Open In App

GATE | GATE-CS-2015 (Set 1) | Question 65

Like Article
Like
Save
Share
Report

The given statement is following by some courses of action. Assuming the statement to be true, decide the correct option.

Statement: 
There has been a significant drop in the water level 
in the lakes supplying water to the city.

Course of action:
1. The water supply authority should impose a partial 
   cut in supply to tackle the situation.
2. The government should appeal to all the residents 
   through mass media for minimal use of water.
3. The government should ban the water supply in lower 
   areas

(A)

Statement 1 and 2 follow

(B)

Statement 1 and 3 follow

(C)

Statement 2 and 3 follow

(D)

All statements follow



Answer: (A)

Explanation:

Here’s a detailed explanation for why option (A) “Statement 1 and 2 follow” is the correct answer:
Statement: There has been a significant drop in the water level in the lakes supplying water to the city.
Course of action: The water supply authority should impose a partial cut in supply to tackle the situation. This course of action is appropriate because when there is a significant drop in the water level in the lakes supplying water to the city, it becomes necessary to conserve and manage the available water resources effectively. Imposing a partial cut in the water supply can help ensure that the available water is distributed efficiently and sustainably.
Course of action: The government should appeal to all the residents through mass media for minimal use of water. This course of action is also reasonable because when there is a significant drop in the water level, it is important to raise awareness among the residents about the need for water conservation. By appealing through mass media, the government can effectively communicate the urgency and encourage residents to use water minimally and responsibly.
Course of action: The government should ban the water supply in lower areas. This course of action is not suitable based on the given statement. The statement mentions a significant drop in the water level in the lakes supplying water to the city, but it does not specify that the situation is critical enough to warrant a complete ban on water supply in specific areas. A complete ban on water supply in lower areas would be a drastic measure and may not be necessary or appropriate at this stage.
Considering the information provided in the statement, options (B), (C), and (D) are not correct because they include course of action 3, which is not supported by the given statement.
Therefore, the correct answer is (A) “Statement 1 and 2 follow” as both courses of action are reasonable and relevant in the given situation


Quiz of this Question
Please comment below if you find anything wrong in the above post


Last Updated : 28 Jun, 2021
Like Article
Save Article
Previous
Next
Share your thoughts in the comments
Similar Reads